1
$\begingroup$

Suppose that I have a set of observations indexed by $i$. Each observation belongs to a group $g$. Let's define by $\hat{Z}_i=Z_i - E\left[Z_i\vert g(i)\right]$ is the residual after projecting $Z$ on the groups $g$.

I have the following system to estimate: $$ Y_i=a+\beta X_i +\varepsilon_i $$ and $$ X_i=b+\alpha \hat{Z}_i + u_i $$ where I know that $E\left[\hat{Z} \varepsilon_i \right]=0$. So I can (I think) estimate $\beta$ by using $\hat{Z}$ as an instrument and where $\hat{Z}$ would be constructed as the residual after a regression of $Z_i$ on a set of groups fixed effects.

I would like to know if I could use $Z$ directly as an instrument instead if I include fixed effects in the first and second stage equations? I.e. if I write the system $$ Y_i=a^0+FE_g+\beta X_i + \varepsilon^0_i $$ and $$ X_i=b^0+FE_g+\alpha^0 Z_i + u^0_i, $$ is it true that $E\left[Z \varepsilon_i \right]=0$? Intuitively it feels like this should work but of course my intuition can be wrong and I would like to have a formal proof of this. Thank you!

$\endgroup$
2
  • $\begingroup$ $\hat{Z}_i=Z_i - E\left[Z_i\vert g(i)\right]$ is not "the orthogonal projection of $Z$ on the groups $g$". It's the (population) residual after projecting. You may want to check your model. $\endgroup$ Commented Aug 28, 2020 at 2:32
  • $\begingroup$ Right, sorry that's what I meant to write. I'll fix it. $\endgroup$ Commented Aug 28, 2020 at 14:00

0

Your Answer

By clicking “Post Your Answer”, you agree to our terms of service and acknowledge you have read our privacy policy.

Start asking to get answers

Find the answer to your question by asking.

Ask question

Explore related questions

See similar questions with these tags.